Asymptotics of the sum $sum_{n=1}^infty frac{x^n}{n^n}$












4














How does the sum
$$f(x)=sum_{n=1}^infty frac{x^n}{n^n}$$
behave asymptotically as $xtoinfty$? It appears that $f(x)$ asymptotically dominates any polynomial and is dominated by any exponential, so we might consider $log f(x)$ rather than $f(x)$.



I apologize for having no work to show on this problem; I have no idea how to begin tackling a problem regarding the asymptotics of a function given its power series (of which there is no hope of evaluating in closed form). Hopefully an answer will provide me with some tools for doing so.










share|cite|improve this question


















  • 4




    why do you think it's dominated by any exponential?
    – zhw.
    2 days ago










  • Using some heuristic reasoning I guess that $$f(x) sim e^{x/e} sqrt{frac{2pi x}{e}}.$$
    – Antonio Vargas
    2 days ago










  • @Frpzzd How accurate do you want your asymptotics to be? I can improve my upper and lower bounds on the asymptotics of $log f$ by the same technique as my current answer. Is it worth it?
    – Clement C.
    yesterday










  • @ClementC. Yes, that would be awesome! It would be nice to know $log f$ within $O(1/x^2)$ or $O(1/x)$, but that might be a stretch... I am interested to see what magic you can work with it, though. :D
    – Frpzzd
    yesterday










  • @Frpzzd A priori, I can mostly improve the constant $c>1/e$ in the upper bound $log f(x) leq cx + o(x)$ (and also improve a bit the $o(x)$ in the lower bound).
    – Clement C.
    yesterday
















4














How does the sum
$$f(x)=sum_{n=1}^infty frac{x^n}{n^n}$$
behave asymptotically as $xtoinfty$? It appears that $f(x)$ asymptotically dominates any polynomial and is dominated by any exponential, so we might consider $log f(x)$ rather than $f(x)$.



I apologize for having no work to show on this problem; I have no idea how to begin tackling a problem regarding the asymptotics of a function given its power series (of which there is no hope of evaluating in closed form). Hopefully an answer will provide me with some tools for doing so.










share|cite|improve this question


















  • 4




    why do you think it's dominated by any exponential?
    – zhw.
    2 days ago










  • Using some heuristic reasoning I guess that $$f(x) sim e^{x/e} sqrt{frac{2pi x}{e}}.$$
    – Antonio Vargas
    2 days ago










  • @Frpzzd How accurate do you want your asymptotics to be? I can improve my upper and lower bounds on the asymptotics of $log f$ by the same technique as my current answer. Is it worth it?
    – Clement C.
    yesterday










  • @ClementC. Yes, that would be awesome! It would be nice to know $log f$ within $O(1/x^2)$ or $O(1/x)$, but that might be a stretch... I am interested to see what magic you can work with it, though. :D
    – Frpzzd
    yesterday










  • @Frpzzd A priori, I can mostly improve the constant $c>1/e$ in the upper bound $log f(x) leq cx + o(x)$ (and also improve a bit the $o(x)$ in the lower bound).
    – Clement C.
    yesterday














4












4








4


2





How does the sum
$$f(x)=sum_{n=1}^infty frac{x^n}{n^n}$$
behave asymptotically as $xtoinfty$? It appears that $f(x)$ asymptotically dominates any polynomial and is dominated by any exponential, so we might consider $log f(x)$ rather than $f(x)$.



I apologize for having no work to show on this problem; I have no idea how to begin tackling a problem regarding the asymptotics of a function given its power series (of which there is no hope of evaluating in closed form). Hopefully an answer will provide me with some tools for doing so.










share|cite|improve this question













How does the sum
$$f(x)=sum_{n=1}^infty frac{x^n}{n^n}$$
behave asymptotically as $xtoinfty$? It appears that $f(x)$ asymptotically dominates any polynomial and is dominated by any exponential, so we might consider $log f(x)$ rather than $f(x)$.



I apologize for having no work to show on this problem; I have no idea how to begin tackling a problem regarding the asymptotics of a function given its power series (of which there is no hope of evaluating in closed form). Hopefully an answer will provide me with some tools for doing so.







sequences-and-series summation asymptotics taylor-expansion






share|cite|improve this question













share|cite|improve this question











share|cite|improve this question




share|cite|improve this question










asked 2 days ago









Frpzzd

21.8k839107




21.8k839107








  • 4




    why do you think it's dominated by any exponential?
    – zhw.
    2 days ago










  • Using some heuristic reasoning I guess that $$f(x) sim e^{x/e} sqrt{frac{2pi x}{e}}.$$
    – Antonio Vargas
    2 days ago










  • @Frpzzd How accurate do you want your asymptotics to be? I can improve my upper and lower bounds on the asymptotics of $log f$ by the same technique as my current answer. Is it worth it?
    – Clement C.
    yesterday










  • @ClementC. Yes, that would be awesome! It would be nice to know $log f$ within $O(1/x^2)$ or $O(1/x)$, but that might be a stretch... I am interested to see what magic you can work with it, though. :D
    – Frpzzd
    yesterday










  • @Frpzzd A priori, I can mostly improve the constant $c>1/e$ in the upper bound $log f(x) leq cx + o(x)$ (and also improve a bit the $o(x)$ in the lower bound).
    – Clement C.
    yesterday














  • 4




    why do you think it's dominated by any exponential?
    – zhw.
    2 days ago










  • Using some heuristic reasoning I guess that $$f(x) sim e^{x/e} sqrt{frac{2pi x}{e}}.$$
    – Antonio Vargas
    2 days ago










  • @Frpzzd How accurate do you want your asymptotics to be? I can improve my upper and lower bounds on the asymptotics of $log f$ by the same technique as my current answer. Is it worth it?
    – Clement C.
    yesterday










  • @ClementC. Yes, that would be awesome! It would be nice to know $log f$ within $O(1/x^2)$ or $O(1/x)$, but that might be a stretch... I am interested to see what magic you can work with it, though. :D
    – Frpzzd
    yesterday










  • @Frpzzd A priori, I can mostly improve the constant $c>1/e$ in the upper bound $log f(x) leq cx + o(x)$ (and also improve a bit the $o(x)$ in the lower bound).
    – Clement C.
    yesterday








4




4




why do you think it's dominated by any exponential?
– zhw.
2 days ago




why do you think it's dominated by any exponential?
– zhw.
2 days ago












Using some heuristic reasoning I guess that $$f(x) sim e^{x/e} sqrt{frac{2pi x}{e}}.$$
– Antonio Vargas
2 days ago




Using some heuristic reasoning I guess that $$f(x) sim e^{x/e} sqrt{frac{2pi x}{e}}.$$
– Antonio Vargas
2 days ago












@Frpzzd How accurate do you want your asymptotics to be? I can improve my upper and lower bounds on the asymptotics of $log f$ by the same technique as my current answer. Is it worth it?
– Clement C.
yesterday




@Frpzzd How accurate do you want your asymptotics to be? I can improve my upper and lower bounds on the asymptotics of $log f$ by the same technique as my current answer. Is it worth it?
– Clement C.
yesterday












@ClementC. Yes, that would be awesome! It would be nice to know $log f$ within $O(1/x^2)$ or $O(1/x)$, but that might be a stretch... I am interested to see what magic you can work with it, though. :D
– Frpzzd
yesterday




@ClementC. Yes, that would be awesome! It would be nice to know $log f$ within $O(1/x^2)$ or $O(1/x)$, but that might be a stretch... I am interested to see what magic you can work with it, though. :D
– Frpzzd
yesterday












@Frpzzd A priori, I can mostly improve the constant $c>1/e$ in the upper bound $log f(x) leq cx + o(x)$ (and also improve a bit the $o(x)$ in the lower bound).
– Clement C.
yesterday




@Frpzzd A priori, I can mostly improve the constant $c>1/e$ in the upper bound $log f(x) leq cx + o(x)$ (and also improve a bit the $o(x)$ in the lower bound).
– Clement C.
yesterday










2 Answers
2






active

oldest

votes


















10














Written from an airport. This is somewhat sketchy when comparing solutions to differential equations, but hopefully not too much for you to fill the gaps.



The main idea: bounding $f$ via differential partial equation.



We have
$$
f'(x) = sum_{n=1}^infty frac{x^{n-1}}{n^{n-1}}
= sum_{n=0}^infty frac{x^{n}}{(n+1)^{n}}
= 1+sum_{n=1}^infty frac{frac{x^{n}}{n^n} }{left(1+frac{1}{n}right)^{n}}
> 1+frac{1}{e}sum_{n=1}^infty frac{x^{n}}{n^n} = 1+frac{1}{e}f(x) tag{1}
$$

so in particular
$$
f' > 1+frac{1}{e}f tag{2}
$$

Since $f(0) = 0$, and the solution to $g' = 1+e^{-1}g$ with $g(0)=0$ is given by $g(x) = e^{x/e+1}-e$, we have
$$
f(x) geq e^{x/e+1}-e > 2e^{x/e} , qquad x>4tag{3}
$$

($x>4$ for the second inequality to kick in).
Now, from $(1)$ we also have
$$
f' < 1+f tag{4}
$$

(we can even improve this to $f' < 1+frac{2}{3}f$), which this time gives
$$
f(x) leq e^x - 1tag{5}
$$



Overall, for $x>4$,
$$
2e^{x/e} leq f(x) leq e^x - 1 tag{6}
$$

which provides a loose estimate of the asymptotic growth of $f$: namely, $boxed{f(x) = e^{Theta(x)}}$.





Further: Improving (slightly) on the lower bound on $log f$ by the low-order terms, and improving on the constant in the main asymptotics of the upper bound of $log f$.



I will show
$$
h(x) leq f(x) leq g(x) tag{7}
$$

where
$$
begin{align}
log h(x) &= frac{1}{e}x + 4 - logfrac{32}{3} + o(1) tag{8}\
log g(x) &= frac{256}{625}x + O(1) tag{9}
end{align}
$$

(note that $frac{256}{625} approx frac{1}{e}+0.04$). Moreover, this can be improved by the same method, pushing to more accurate estimates, but this will get uglier. (One can also push the Taylor expansion above further, based on (12) and (13). I stopped at $o(1)$).



The observation is that for the upper and lower bound, we bounded uniformly the coefficients by
$$
forall n geq 1, qquad frac{1}{n^n} leq frac{1}{left(1+frac{1}{n}right)^{n}} cdot frac{1}{n^n} leq frac{1}{e}cdot frac{1}{n^n}
$$

to obtain the two corresponding differential equations. We can do better by handling the first few terms more tightly. Namely, we have
$$
left(1+frac{1}{n}right)^n = begin{cases}
frac{1}{2} & n=1\
frac{4}{9} & n=2\
frac{27}{64} & n=3\
frac{256}{625} & n=4
end{cases}
$$

(and, of course, $left(1+frac{1}{n}right)^n$ is decreasing to $1/e$). Thus, we can leverage this and solve instead the following two differential equations for $h$ and $g$:
begin{align}
h'(x) &= 1 + left(frac{1}{2} - frac{1}{e}right) x + left(frac{4}{9} - frac{1}{e}right) frac{x^2}{4} + left( frac{27}{64} - frac{1}{e}right) frac{x^3}{27} + frac{1}{e}h(x)tag{10}\
g'(x) &= 1 + left(frac{1}{2} - frac{256}{625}right) x + left(frac{4}{9} - frac{256}{625}right) frac{x^2}{4} + left( frac{27}{64} - frac{256}{625}right) frac{x^3}{27} + frac{256}{625}g(x)tag{11}
end{align}

subject to $h(0)=g(0)=0$. Solving those gives a nasty expression,
begin{align}
h(x) &= frac{3}{32} e^{4 + frac{1}{e}x}
+ left(frac{1}{27} - frac{e}{64}right) x^3
+ left(frac{1}{4} - frac{3 e^2}{64}right) x^2
+ left(1 - frac{3e^3}{32}right) x
-frac{3e^4}{32} tag{12} \
g(x) &= frac{457763671875}{137438953472}e^{frac{256}{625}x}
- frac{491}{442368}x^3
- frac{123299}{4194304}x^2
- frac{195550963}{536870912}x
-frac{457763671875}{137438953472} tag{13} \
end{align}

leading to the claimed (8) and (9).



Below, a plot illustrating those approximations:



enter image description here






share|cite|improve this answer























  • Note: I believe the lower bound to be tight, i.e., $log f(x) = x/e +o(x)$. One can get closer to the constant $1/e$ by the above approach, by treating the first few terms of the series separately to get a better differential equation lower bound (since higher terms allow a better bound on $(1+1/n)^n$).
    – Clement C.
    2 days ago












  • Note 2: actually, f(0)=0. This won't change much and the conclusion stands, but the answer needs to be edited. I'll do it when landing.
    – Clement C.
    2 days ago










  • "Please make sure your table is in the upright position. We'll be taking off shortly... "No!!! But Frpzzd will enjoy this explanation so much more if there is a graph!!!" "Sir... please put the laptop away. "
    – Mason
    2 days ago






  • 1




    OR take $g(x) = frac{f(x)}{e^{x/e}},$ where you have proved that $g' > 0$
    – Will Jagy
    2 days ago










  • +1. Good answer. I am going to see if we can use Abel's summation formula to corroborate this work. If I find a way of doing that I will try and write it up.
    – Mason
    2 days ago



















4














There is an analogue of Laplace's method which works for sums. $n ln(x/n)$ attains the maximum at $n = x/e$. Writing the exponent as $n ln(x/n) = x/e - x xi^2$, computing the expansion of $n'(xi)$ at $xi = 0$ and extending the integration range to $(-infty, infty)$, we obtain
$$frac {n'(xi)} x =
sqrt{frac 2 e} + c_1 xi -
frac 1 6 sqrt{frac e 2} ,xi^2 + c_3 xi^3 + O(xi^4),
quad xi to 0,\
sum_{n geq 1} frac {x^n} {n^n} =
int_{-infty}^infty
x left( sqrt{frac 2 e} - frac 1 6 sqrt{frac e 2} ,xi^2 +
O(xi^4) right)
e^{x/e - x xi^2} dxi = \
sqrt{frac pi 2} ,e^{x/e} left(
2sqrt{frac x e} - frac 1 {12} sqrt{frac e x} + O(x^{-3/2}) right),
quad x to infty,$$

which gives $ln f(x)$ with an error of order $O(x^{-2})$.






share|cite|improve this answer























    Your Answer





    StackExchange.ifUsing("editor", function () {
    return StackExchange.using("mathjaxEditing", function () {
    StackExchange.MarkdownEditor.creationCallbacks.add(function (editor, postfix) {
    StackExchange.mathjaxEditing.prepareWmdForMathJax(editor, postfix, [["$", "$"], ["\\(","\\)"]]);
    });
    });
    }, "mathjax-editing");

    StackExchange.ready(function() {
    var channelOptions = {
    tags: "".split(" "),
    id: "69"
    };
    initTagRenderer("".split(" "), "".split(" "), channelOptions);

    StackExchange.using("externalEditor", function() {
    // Have to fire editor after snippets, if snippets enabled
    if (StackExchange.settings.snippets.snippetsEnabled) {
    StackExchange.using("snippets", function() {
    createEditor();
    });
    }
    else {
    createEditor();
    }
    });

    function createEditor() {
    StackExchange.prepareEditor({
    heartbeatType: 'answer',
    autoActivateHeartbeat: false,
    convertImagesToLinks: true,
    noModals: true,
    showLowRepImageUploadWarning: true,
    reputationToPostImages: 10,
    bindNavPrevention: true,
    postfix: "",
    imageUploader: {
    brandingHtml: "Powered by u003ca class="icon-imgur-white" href="https://imgur.com/"u003eu003c/au003e",
    contentPolicyHtml: "User contributions licensed under u003ca href="https://creativecommons.org/licenses/by-sa/3.0/"u003ecc by-sa 3.0 with attribution requiredu003c/au003e u003ca href="https://stackoverflow.com/legal/content-policy"u003e(content policy)u003c/au003e",
    allowUrls: true
    },
    noCode: true, onDemand: true,
    discardSelector: ".discard-answer"
    ,immediatelyShowMarkdownHelp:true
    });


    }
    });














    draft saved

    draft discarded


















    StackExchange.ready(
    function () {
    StackExchange.openid.initPostLogin('.new-post-login', 'https%3a%2f%2fmath.stackexchange.com%2fquestions%2f3052328%2fasymptotics-of-the-sum-sum-n-1-infty-fracxnnn%23new-answer', 'question_page');
    }
    );

    Post as a guest















    Required, but never shown

























    2 Answers
    2






    active

    oldest

    votes








    2 Answers
    2






    active

    oldest

    votes









    active

    oldest

    votes






    active

    oldest

    votes









    10














    Written from an airport. This is somewhat sketchy when comparing solutions to differential equations, but hopefully not too much for you to fill the gaps.



    The main idea: bounding $f$ via differential partial equation.



    We have
    $$
    f'(x) = sum_{n=1}^infty frac{x^{n-1}}{n^{n-1}}
    = sum_{n=0}^infty frac{x^{n}}{(n+1)^{n}}
    = 1+sum_{n=1}^infty frac{frac{x^{n}}{n^n} }{left(1+frac{1}{n}right)^{n}}
    > 1+frac{1}{e}sum_{n=1}^infty frac{x^{n}}{n^n} = 1+frac{1}{e}f(x) tag{1}
    $$

    so in particular
    $$
    f' > 1+frac{1}{e}f tag{2}
    $$

    Since $f(0) = 0$, and the solution to $g' = 1+e^{-1}g$ with $g(0)=0$ is given by $g(x) = e^{x/e+1}-e$, we have
    $$
    f(x) geq e^{x/e+1}-e > 2e^{x/e} , qquad x>4tag{3}
    $$

    ($x>4$ for the second inequality to kick in).
    Now, from $(1)$ we also have
    $$
    f' < 1+f tag{4}
    $$

    (we can even improve this to $f' < 1+frac{2}{3}f$), which this time gives
    $$
    f(x) leq e^x - 1tag{5}
    $$



    Overall, for $x>4$,
    $$
    2e^{x/e} leq f(x) leq e^x - 1 tag{6}
    $$

    which provides a loose estimate of the asymptotic growth of $f$: namely, $boxed{f(x) = e^{Theta(x)}}$.





    Further: Improving (slightly) on the lower bound on $log f$ by the low-order terms, and improving on the constant in the main asymptotics of the upper bound of $log f$.



    I will show
    $$
    h(x) leq f(x) leq g(x) tag{7}
    $$

    where
    $$
    begin{align}
    log h(x) &= frac{1}{e}x + 4 - logfrac{32}{3} + o(1) tag{8}\
    log g(x) &= frac{256}{625}x + O(1) tag{9}
    end{align}
    $$

    (note that $frac{256}{625} approx frac{1}{e}+0.04$). Moreover, this can be improved by the same method, pushing to more accurate estimates, but this will get uglier. (One can also push the Taylor expansion above further, based on (12) and (13). I stopped at $o(1)$).



    The observation is that for the upper and lower bound, we bounded uniformly the coefficients by
    $$
    forall n geq 1, qquad frac{1}{n^n} leq frac{1}{left(1+frac{1}{n}right)^{n}} cdot frac{1}{n^n} leq frac{1}{e}cdot frac{1}{n^n}
    $$

    to obtain the two corresponding differential equations. We can do better by handling the first few terms more tightly. Namely, we have
    $$
    left(1+frac{1}{n}right)^n = begin{cases}
    frac{1}{2} & n=1\
    frac{4}{9} & n=2\
    frac{27}{64} & n=3\
    frac{256}{625} & n=4
    end{cases}
    $$

    (and, of course, $left(1+frac{1}{n}right)^n$ is decreasing to $1/e$). Thus, we can leverage this and solve instead the following two differential equations for $h$ and $g$:
    begin{align}
    h'(x) &= 1 + left(frac{1}{2} - frac{1}{e}right) x + left(frac{4}{9} - frac{1}{e}right) frac{x^2}{4} + left( frac{27}{64} - frac{1}{e}right) frac{x^3}{27} + frac{1}{e}h(x)tag{10}\
    g'(x) &= 1 + left(frac{1}{2} - frac{256}{625}right) x + left(frac{4}{9} - frac{256}{625}right) frac{x^2}{4} + left( frac{27}{64} - frac{256}{625}right) frac{x^3}{27} + frac{256}{625}g(x)tag{11}
    end{align}

    subject to $h(0)=g(0)=0$. Solving those gives a nasty expression,
    begin{align}
    h(x) &= frac{3}{32} e^{4 + frac{1}{e}x}
    + left(frac{1}{27} - frac{e}{64}right) x^3
    + left(frac{1}{4} - frac{3 e^2}{64}right) x^2
    + left(1 - frac{3e^3}{32}right) x
    -frac{3e^4}{32} tag{12} \
    g(x) &= frac{457763671875}{137438953472}e^{frac{256}{625}x}
    - frac{491}{442368}x^3
    - frac{123299}{4194304}x^2
    - frac{195550963}{536870912}x
    -frac{457763671875}{137438953472} tag{13} \
    end{align}

    leading to the claimed (8) and (9).



    Below, a plot illustrating those approximations:



    enter image description here






    share|cite|improve this answer























    • Note: I believe the lower bound to be tight, i.e., $log f(x) = x/e +o(x)$. One can get closer to the constant $1/e$ by the above approach, by treating the first few terms of the series separately to get a better differential equation lower bound (since higher terms allow a better bound on $(1+1/n)^n$).
      – Clement C.
      2 days ago












    • Note 2: actually, f(0)=0. This won't change much and the conclusion stands, but the answer needs to be edited. I'll do it when landing.
      – Clement C.
      2 days ago










    • "Please make sure your table is in the upright position. We'll be taking off shortly... "No!!! But Frpzzd will enjoy this explanation so much more if there is a graph!!!" "Sir... please put the laptop away. "
      – Mason
      2 days ago






    • 1




      OR take $g(x) = frac{f(x)}{e^{x/e}},$ where you have proved that $g' > 0$
      – Will Jagy
      2 days ago










    • +1. Good answer. I am going to see if we can use Abel's summation formula to corroborate this work. If I find a way of doing that I will try and write it up.
      – Mason
      2 days ago
















    10














    Written from an airport. This is somewhat sketchy when comparing solutions to differential equations, but hopefully not too much for you to fill the gaps.



    The main idea: bounding $f$ via differential partial equation.



    We have
    $$
    f'(x) = sum_{n=1}^infty frac{x^{n-1}}{n^{n-1}}
    = sum_{n=0}^infty frac{x^{n}}{(n+1)^{n}}
    = 1+sum_{n=1}^infty frac{frac{x^{n}}{n^n} }{left(1+frac{1}{n}right)^{n}}
    > 1+frac{1}{e}sum_{n=1}^infty frac{x^{n}}{n^n} = 1+frac{1}{e}f(x) tag{1}
    $$

    so in particular
    $$
    f' > 1+frac{1}{e}f tag{2}
    $$

    Since $f(0) = 0$, and the solution to $g' = 1+e^{-1}g$ with $g(0)=0$ is given by $g(x) = e^{x/e+1}-e$, we have
    $$
    f(x) geq e^{x/e+1}-e > 2e^{x/e} , qquad x>4tag{3}
    $$

    ($x>4$ for the second inequality to kick in).
    Now, from $(1)$ we also have
    $$
    f' < 1+f tag{4}
    $$

    (we can even improve this to $f' < 1+frac{2}{3}f$), which this time gives
    $$
    f(x) leq e^x - 1tag{5}
    $$



    Overall, for $x>4$,
    $$
    2e^{x/e} leq f(x) leq e^x - 1 tag{6}
    $$

    which provides a loose estimate of the asymptotic growth of $f$: namely, $boxed{f(x) = e^{Theta(x)}}$.





    Further: Improving (slightly) on the lower bound on $log f$ by the low-order terms, and improving on the constant in the main asymptotics of the upper bound of $log f$.



    I will show
    $$
    h(x) leq f(x) leq g(x) tag{7}
    $$

    where
    $$
    begin{align}
    log h(x) &= frac{1}{e}x + 4 - logfrac{32}{3} + o(1) tag{8}\
    log g(x) &= frac{256}{625}x + O(1) tag{9}
    end{align}
    $$

    (note that $frac{256}{625} approx frac{1}{e}+0.04$). Moreover, this can be improved by the same method, pushing to more accurate estimates, but this will get uglier. (One can also push the Taylor expansion above further, based on (12) and (13). I stopped at $o(1)$).



    The observation is that for the upper and lower bound, we bounded uniformly the coefficients by
    $$
    forall n geq 1, qquad frac{1}{n^n} leq frac{1}{left(1+frac{1}{n}right)^{n}} cdot frac{1}{n^n} leq frac{1}{e}cdot frac{1}{n^n}
    $$

    to obtain the two corresponding differential equations. We can do better by handling the first few terms more tightly. Namely, we have
    $$
    left(1+frac{1}{n}right)^n = begin{cases}
    frac{1}{2} & n=1\
    frac{4}{9} & n=2\
    frac{27}{64} & n=3\
    frac{256}{625} & n=4
    end{cases}
    $$

    (and, of course, $left(1+frac{1}{n}right)^n$ is decreasing to $1/e$). Thus, we can leverage this and solve instead the following two differential equations for $h$ and $g$:
    begin{align}
    h'(x) &= 1 + left(frac{1}{2} - frac{1}{e}right) x + left(frac{4}{9} - frac{1}{e}right) frac{x^2}{4} + left( frac{27}{64} - frac{1}{e}right) frac{x^3}{27} + frac{1}{e}h(x)tag{10}\
    g'(x) &= 1 + left(frac{1}{2} - frac{256}{625}right) x + left(frac{4}{9} - frac{256}{625}right) frac{x^2}{4} + left( frac{27}{64} - frac{256}{625}right) frac{x^3}{27} + frac{256}{625}g(x)tag{11}
    end{align}

    subject to $h(0)=g(0)=0$. Solving those gives a nasty expression,
    begin{align}
    h(x) &= frac{3}{32} e^{4 + frac{1}{e}x}
    + left(frac{1}{27} - frac{e}{64}right) x^3
    + left(frac{1}{4} - frac{3 e^2}{64}right) x^2
    + left(1 - frac{3e^3}{32}right) x
    -frac{3e^4}{32} tag{12} \
    g(x) &= frac{457763671875}{137438953472}e^{frac{256}{625}x}
    - frac{491}{442368}x^3
    - frac{123299}{4194304}x^2
    - frac{195550963}{536870912}x
    -frac{457763671875}{137438953472} tag{13} \
    end{align}

    leading to the claimed (8) and (9).



    Below, a plot illustrating those approximations:



    enter image description here






    share|cite|improve this answer























    • Note: I believe the lower bound to be tight, i.e., $log f(x) = x/e +o(x)$. One can get closer to the constant $1/e$ by the above approach, by treating the first few terms of the series separately to get a better differential equation lower bound (since higher terms allow a better bound on $(1+1/n)^n$).
      – Clement C.
      2 days ago












    • Note 2: actually, f(0)=0. This won't change much and the conclusion stands, but the answer needs to be edited. I'll do it when landing.
      – Clement C.
      2 days ago










    • "Please make sure your table is in the upright position. We'll be taking off shortly... "No!!! But Frpzzd will enjoy this explanation so much more if there is a graph!!!" "Sir... please put the laptop away. "
      – Mason
      2 days ago






    • 1




      OR take $g(x) = frac{f(x)}{e^{x/e}},$ where you have proved that $g' > 0$
      – Will Jagy
      2 days ago










    • +1. Good answer. I am going to see if we can use Abel's summation formula to corroborate this work. If I find a way of doing that I will try and write it up.
      – Mason
      2 days ago














    10












    10








    10






    Written from an airport. This is somewhat sketchy when comparing solutions to differential equations, but hopefully not too much for you to fill the gaps.



    The main idea: bounding $f$ via differential partial equation.



    We have
    $$
    f'(x) = sum_{n=1}^infty frac{x^{n-1}}{n^{n-1}}
    = sum_{n=0}^infty frac{x^{n}}{(n+1)^{n}}
    = 1+sum_{n=1}^infty frac{frac{x^{n}}{n^n} }{left(1+frac{1}{n}right)^{n}}
    > 1+frac{1}{e}sum_{n=1}^infty frac{x^{n}}{n^n} = 1+frac{1}{e}f(x) tag{1}
    $$

    so in particular
    $$
    f' > 1+frac{1}{e}f tag{2}
    $$

    Since $f(0) = 0$, and the solution to $g' = 1+e^{-1}g$ with $g(0)=0$ is given by $g(x) = e^{x/e+1}-e$, we have
    $$
    f(x) geq e^{x/e+1}-e > 2e^{x/e} , qquad x>4tag{3}
    $$

    ($x>4$ for the second inequality to kick in).
    Now, from $(1)$ we also have
    $$
    f' < 1+f tag{4}
    $$

    (we can even improve this to $f' < 1+frac{2}{3}f$), which this time gives
    $$
    f(x) leq e^x - 1tag{5}
    $$



    Overall, for $x>4$,
    $$
    2e^{x/e} leq f(x) leq e^x - 1 tag{6}
    $$

    which provides a loose estimate of the asymptotic growth of $f$: namely, $boxed{f(x) = e^{Theta(x)}}$.





    Further: Improving (slightly) on the lower bound on $log f$ by the low-order terms, and improving on the constant in the main asymptotics of the upper bound of $log f$.



    I will show
    $$
    h(x) leq f(x) leq g(x) tag{7}
    $$

    where
    $$
    begin{align}
    log h(x) &= frac{1}{e}x + 4 - logfrac{32}{3} + o(1) tag{8}\
    log g(x) &= frac{256}{625}x + O(1) tag{9}
    end{align}
    $$

    (note that $frac{256}{625} approx frac{1}{e}+0.04$). Moreover, this can be improved by the same method, pushing to more accurate estimates, but this will get uglier. (One can also push the Taylor expansion above further, based on (12) and (13). I stopped at $o(1)$).



    The observation is that for the upper and lower bound, we bounded uniformly the coefficients by
    $$
    forall n geq 1, qquad frac{1}{n^n} leq frac{1}{left(1+frac{1}{n}right)^{n}} cdot frac{1}{n^n} leq frac{1}{e}cdot frac{1}{n^n}
    $$

    to obtain the two corresponding differential equations. We can do better by handling the first few terms more tightly. Namely, we have
    $$
    left(1+frac{1}{n}right)^n = begin{cases}
    frac{1}{2} & n=1\
    frac{4}{9} & n=2\
    frac{27}{64} & n=3\
    frac{256}{625} & n=4
    end{cases}
    $$

    (and, of course, $left(1+frac{1}{n}right)^n$ is decreasing to $1/e$). Thus, we can leverage this and solve instead the following two differential equations for $h$ and $g$:
    begin{align}
    h'(x) &= 1 + left(frac{1}{2} - frac{1}{e}right) x + left(frac{4}{9} - frac{1}{e}right) frac{x^2}{4} + left( frac{27}{64} - frac{1}{e}right) frac{x^3}{27} + frac{1}{e}h(x)tag{10}\
    g'(x) &= 1 + left(frac{1}{2} - frac{256}{625}right) x + left(frac{4}{9} - frac{256}{625}right) frac{x^2}{4} + left( frac{27}{64} - frac{256}{625}right) frac{x^3}{27} + frac{256}{625}g(x)tag{11}
    end{align}

    subject to $h(0)=g(0)=0$. Solving those gives a nasty expression,
    begin{align}
    h(x) &= frac{3}{32} e^{4 + frac{1}{e}x}
    + left(frac{1}{27} - frac{e}{64}right) x^3
    + left(frac{1}{4} - frac{3 e^2}{64}right) x^2
    + left(1 - frac{3e^3}{32}right) x
    -frac{3e^4}{32} tag{12} \
    g(x) &= frac{457763671875}{137438953472}e^{frac{256}{625}x}
    - frac{491}{442368}x^3
    - frac{123299}{4194304}x^2
    - frac{195550963}{536870912}x
    -frac{457763671875}{137438953472} tag{13} \
    end{align}

    leading to the claimed (8) and (9).



    Below, a plot illustrating those approximations:



    enter image description here






    share|cite|improve this answer














    Written from an airport. This is somewhat sketchy when comparing solutions to differential equations, but hopefully not too much for you to fill the gaps.



    The main idea: bounding $f$ via differential partial equation.



    We have
    $$
    f'(x) = sum_{n=1}^infty frac{x^{n-1}}{n^{n-1}}
    = sum_{n=0}^infty frac{x^{n}}{(n+1)^{n}}
    = 1+sum_{n=1}^infty frac{frac{x^{n}}{n^n} }{left(1+frac{1}{n}right)^{n}}
    > 1+frac{1}{e}sum_{n=1}^infty frac{x^{n}}{n^n} = 1+frac{1}{e}f(x) tag{1}
    $$

    so in particular
    $$
    f' > 1+frac{1}{e}f tag{2}
    $$

    Since $f(0) = 0$, and the solution to $g' = 1+e^{-1}g$ with $g(0)=0$ is given by $g(x) = e^{x/e+1}-e$, we have
    $$
    f(x) geq e^{x/e+1}-e > 2e^{x/e} , qquad x>4tag{3}
    $$

    ($x>4$ for the second inequality to kick in).
    Now, from $(1)$ we also have
    $$
    f' < 1+f tag{4}
    $$

    (we can even improve this to $f' < 1+frac{2}{3}f$), which this time gives
    $$
    f(x) leq e^x - 1tag{5}
    $$



    Overall, for $x>4$,
    $$
    2e^{x/e} leq f(x) leq e^x - 1 tag{6}
    $$

    which provides a loose estimate of the asymptotic growth of $f$: namely, $boxed{f(x) = e^{Theta(x)}}$.





    Further: Improving (slightly) on the lower bound on $log f$ by the low-order terms, and improving on the constant in the main asymptotics of the upper bound of $log f$.



    I will show
    $$
    h(x) leq f(x) leq g(x) tag{7}
    $$

    where
    $$
    begin{align}
    log h(x) &= frac{1}{e}x + 4 - logfrac{32}{3} + o(1) tag{8}\
    log g(x) &= frac{256}{625}x + O(1) tag{9}
    end{align}
    $$

    (note that $frac{256}{625} approx frac{1}{e}+0.04$). Moreover, this can be improved by the same method, pushing to more accurate estimates, but this will get uglier. (One can also push the Taylor expansion above further, based on (12) and (13). I stopped at $o(1)$).



    The observation is that for the upper and lower bound, we bounded uniformly the coefficients by
    $$
    forall n geq 1, qquad frac{1}{n^n} leq frac{1}{left(1+frac{1}{n}right)^{n}} cdot frac{1}{n^n} leq frac{1}{e}cdot frac{1}{n^n}
    $$

    to obtain the two corresponding differential equations. We can do better by handling the first few terms more tightly. Namely, we have
    $$
    left(1+frac{1}{n}right)^n = begin{cases}
    frac{1}{2} & n=1\
    frac{4}{9} & n=2\
    frac{27}{64} & n=3\
    frac{256}{625} & n=4
    end{cases}
    $$

    (and, of course, $left(1+frac{1}{n}right)^n$ is decreasing to $1/e$). Thus, we can leverage this and solve instead the following two differential equations for $h$ and $g$:
    begin{align}
    h'(x) &= 1 + left(frac{1}{2} - frac{1}{e}right) x + left(frac{4}{9} - frac{1}{e}right) frac{x^2}{4} + left( frac{27}{64} - frac{1}{e}right) frac{x^3}{27} + frac{1}{e}h(x)tag{10}\
    g'(x) &= 1 + left(frac{1}{2} - frac{256}{625}right) x + left(frac{4}{9} - frac{256}{625}right) frac{x^2}{4} + left( frac{27}{64} - frac{256}{625}right) frac{x^3}{27} + frac{256}{625}g(x)tag{11}
    end{align}

    subject to $h(0)=g(0)=0$. Solving those gives a nasty expression,
    begin{align}
    h(x) &= frac{3}{32} e^{4 + frac{1}{e}x}
    + left(frac{1}{27} - frac{e}{64}right) x^3
    + left(frac{1}{4} - frac{3 e^2}{64}right) x^2
    + left(1 - frac{3e^3}{32}right) x
    -frac{3e^4}{32} tag{12} \
    g(x) &= frac{457763671875}{137438953472}e^{frac{256}{625}x}
    - frac{491}{442368}x^3
    - frac{123299}{4194304}x^2
    - frac{195550963}{536870912}x
    -frac{457763671875}{137438953472} tag{13} \
    end{align}

    leading to the claimed (8) and (9).



    Below, a plot illustrating those approximations:



    enter image description here







    share|cite|improve this answer














    share|cite|improve this answer



    share|cite|improve this answer








    edited yesterday

























    answered 2 days ago









    Clement C.

    49.6k33886




    49.6k33886












    • Note: I believe the lower bound to be tight, i.e., $log f(x) = x/e +o(x)$. One can get closer to the constant $1/e$ by the above approach, by treating the first few terms of the series separately to get a better differential equation lower bound (since higher terms allow a better bound on $(1+1/n)^n$).
      – Clement C.
      2 days ago












    • Note 2: actually, f(0)=0. This won't change much and the conclusion stands, but the answer needs to be edited. I'll do it when landing.
      – Clement C.
      2 days ago










    • "Please make sure your table is in the upright position. We'll be taking off shortly... "No!!! But Frpzzd will enjoy this explanation so much more if there is a graph!!!" "Sir... please put the laptop away. "
      – Mason
      2 days ago






    • 1




      OR take $g(x) = frac{f(x)}{e^{x/e}},$ where you have proved that $g' > 0$
      – Will Jagy
      2 days ago










    • +1. Good answer. I am going to see if we can use Abel's summation formula to corroborate this work. If I find a way of doing that I will try and write it up.
      – Mason
      2 days ago


















    • Note: I believe the lower bound to be tight, i.e., $log f(x) = x/e +o(x)$. One can get closer to the constant $1/e$ by the above approach, by treating the first few terms of the series separately to get a better differential equation lower bound (since higher terms allow a better bound on $(1+1/n)^n$).
      – Clement C.
      2 days ago












    • Note 2: actually, f(0)=0. This won't change much and the conclusion stands, but the answer needs to be edited. I'll do it when landing.
      – Clement C.
      2 days ago










    • "Please make sure your table is in the upright position. We'll be taking off shortly... "No!!! But Frpzzd will enjoy this explanation so much more if there is a graph!!!" "Sir... please put the laptop away. "
      – Mason
      2 days ago






    • 1




      OR take $g(x) = frac{f(x)}{e^{x/e}},$ where you have proved that $g' > 0$
      – Will Jagy
      2 days ago










    • +1. Good answer. I am going to see if we can use Abel's summation formula to corroborate this work. If I find a way of doing that I will try and write it up.
      – Mason
      2 days ago
















    Note: I believe the lower bound to be tight, i.e., $log f(x) = x/e +o(x)$. One can get closer to the constant $1/e$ by the above approach, by treating the first few terms of the series separately to get a better differential equation lower bound (since higher terms allow a better bound on $(1+1/n)^n$).
    – Clement C.
    2 days ago






    Note: I believe the lower bound to be tight, i.e., $log f(x) = x/e +o(x)$. One can get closer to the constant $1/e$ by the above approach, by treating the first few terms of the series separately to get a better differential equation lower bound (since higher terms allow a better bound on $(1+1/n)^n$).
    – Clement C.
    2 days ago














    Note 2: actually, f(0)=0. This won't change much and the conclusion stands, but the answer needs to be edited. I'll do it when landing.
    – Clement C.
    2 days ago




    Note 2: actually, f(0)=0. This won't change much and the conclusion stands, but the answer needs to be edited. I'll do it when landing.
    – Clement C.
    2 days ago












    "Please make sure your table is in the upright position. We'll be taking off shortly... "No!!! But Frpzzd will enjoy this explanation so much more if there is a graph!!!" "Sir... please put the laptop away. "
    – Mason
    2 days ago




    "Please make sure your table is in the upright position. We'll be taking off shortly... "No!!! But Frpzzd will enjoy this explanation so much more if there is a graph!!!" "Sir... please put the laptop away. "
    – Mason
    2 days ago




    1




    1




    OR take $g(x) = frac{f(x)}{e^{x/e}},$ where you have proved that $g' > 0$
    – Will Jagy
    2 days ago




    OR take $g(x) = frac{f(x)}{e^{x/e}},$ where you have proved that $g' > 0$
    – Will Jagy
    2 days ago












    +1. Good answer. I am going to see if we can use Abel's summation formula to corroborate this work. If I find a way of doing that I will try and write it up.
    – Mason
    2 days ago




    +1. Good answer. I am going to see if we can use Abel's summation formula to corroborate this work. If I find a way of doing that I will try and write it up.
    – Mason
    2 days ago











    4














    There is an analogue of Laplace's method which works for sums. $n ln(x/n)$ attains the maximum at $n = x/e$. Writing the exponent as $n ln(x/n) = x/e - x xi^2$, computing the expansion of $n'(xi)$ at $xi = 0$ and extending the integration range to $(-infty, infty)$, we obtain
    $$frac {n'(xi)} x =
    sqrt{frac 2 e} + c_1 xi -
    frac 1 6 sqrt{frac e 2} ,xi^2 + c_3 xi^3 + O(xi^4),
    quad xi to 0,\
    sum_{n geq 1} frac {x^n} {n^n} =
    int_{-infty}^infty
    x left( sqrt{frac 2 e} - frac 1 6 sqrt{frac e 2} ,xi^2 +
    O(xi^4) right)
    e^{x/e - x xi^2} dxi = \
    sqrt{frac pi 2} ,e^{x/e} left(
    2sqrt{frac x e} - frac 1 {12} sqrt{frac e x} + O(x^{-3/2}) right),
    quad x to infty,$$

    which gives $ln f(x)$ with an error of order $O(x^{-2})$.






    share|cite|improve this answer




























      4














      There is an analogue of Laplace's method which works for sums. $n ln(x/n)$ attains the maximum at $n = x/e$. Writing the exponent as $n ln(x/n) = x/e - x xi^2$, computing the expansion of $n'(xi)$ at $xi = 0$ and extending the integration range to $(-infty, infty)$, we obtain
      $$frac {n'(xi)} x =
      sqrt{frac 2 e} + c_1 xi -
      frac 1 6 sqrt{frac e 2} ,xi^2 + c_3 xi^3 + O(xi^4),
      quad xi to 0,\
      sum_{n geq 1} frac {x^n} {n^n} =
      int_{-infty}^infty
      x left( sqrt{frac 2 e} - frac 1 6 sqrt{frac e 2} ,xi^2 +
      O(xi^4) right)
      e^{x/e - x xi^2} dxi = \
      sqrt{frac pi 2} ,e^{x/e} left(
      2sqrt{frac x e} - frac 1 {12} sqrt{frac e x} + O(x^{-3/2}) right),
      quad x to infty,$$

      which gives $ln f(x)$ with an error of order $O(x^{-2})$.






      share|cite|improve this answer


























        4












        4








        4






        There is an analogue of Laplace's method which works for sums. $n ln(x/n)$ attains the maximum at $n = x/e$. Writing the exponent as $n ln(x/n) = x/e - x xi^2$, computing the expansion of $n'(xi)$ at $xi = 0$ and extending the integration range to $(-infty, infty)$, we obtain
        $$frac {n'(xi)} x =
        sqrt{frac 2 e} + c_1 xi -
        frac 1 6 sqrt{frac e 2} ,xi^2 + c_3 xi^3 + O(xi^4),
        quad xi to 0,\
        sum_{n geq 1} frac {x^n} {n^n} =
        int_{-infty}^infty
        x left( sqrt{frac 2 e} - frac 1 6 sqrt{frac e 2} ,xi^2 +
        O(xi^4) right)
        e^{x/e - x xi^2} dxi = \
        sqrt{frac pi 2} ,e^{x/e} left(
        2sqrt{frac x e} - frac 1 {12} sqrt{frac e x} + O(x^{-3/2}) right),
        quad x to infty,$$

        which gives $ln f(x)$ with an error of order $O(x^{-2})$.






        share|cite|improve this answer














        There is an analogue of Laplace's method which works for sums. $n ln(x/n)$ attains the maximum at $n = x/e$. Writing the exponent as $n ln(x/n) = x/e - x xi^2$, computing the expansion of $n'(xi)$ at $xi = 0$ and extending the integration range to $(-infty, infty)$, we obtain
        $$frac {n'(xi)} x =
        sqrt{frac 2 e} + c_1 xi -
        frac 1 6 sqrt{frac e 2} ,xi^2 + c_3 xi^3 + O(xi^4),
        quad xi to 0,\
        sum_{n geq 1} frac {x^n} {n^n} =
        int_{-infty}^infty
        x left( sqrt{frac 2 e} - frac 1 6 sqrt{frac e 2} ,xi^2 +
        O(xi^4) right)
        e^{x/e - x xi^2} dxi = \
        sqrt{frac pi 2} ,e^{x/e} left(
        2sqrt{frac x e} - frac 1 {12} sqrt{frac e x} + O(x^{-3/2}) right),
        quad x to infty,$$

        which gives $ln f(x)$ with an error of order $O(x^{-2})$.







        share|cite|improve this answer














        share|cite|improve this answer



        share|cite|improve this answer








        edited yesterday

























        answered yesterday









        Maxim

        4,4931219




        4,4931219






























            draft saved

            draft discarded




















































            Thanks for contributing an answer to Mathematics Stack Exchange!


            • Please be sure to answer the question. Provide details and share your research!

            But avoid



            • Asking for help, clarification, or responding to other answers.

            • Making statements based on opinion; back them up with references or personal experience.


            Use MathJax to format equations. MathJax reference.


            To learn more, see our tips on writing great answers.





            Some of your past answers have not been well-received, and you're in danger of being blocked from answering.


            Please pay close attention to the following guidance:


            • Please be sure to answer the question. Provide details and share your research!

            But avoid



            • Asking for help, clarification, or responding to other answers.

            • Making statements based on opinion; back them up with references or personal experience.


            To learn more, see our tips on writing great answers.




            draft saved


            draft discarded














            StackExchange.ready(
            function () {
            StackExchange.openid.initPostLogin('.new-post-login', 'https%3a%2f%2fmath.stackexchange.com%2fquestions%2f3052328%2fasymptotics-of-the-sum-sum-n-1-infty-fracxnnn%23new-answer', 'question_page');
            }
            );

            Post as a guest















            Required, but never shown





















































            Required, but never shown














            Required, but never shown












            Required, but never shown







            Required, but never shown

































            Required, but never shown














            Required, but never shown












            Required, but never shown







            Required, but never shown







            Popular posts from this blog

            Human spaceflight

            Can not write log (Is /dev/pts mounted?) - openpty in Ubuntu-on-Windows?

            File:DeusFollowingSea.jpg